Proof by Induction - New to proofsDominoes and induction, or how does induction work?Proving that...

Incompressible fluid definition

Can the Count of Monte Cristo's calculation of poison dosage be explained?

How to avoid being sexist when trying to employ someone to function in a very sexist environment?

Why does the DC-9-80 have this cusp in its fuselage?

Eww, those bytes are gross

F1 visa even for a three-week course?

Why zero tolerance on nudity in space?

What is the purpose of easy combat scenarios that don't need resource expenditure?

Table enclosed in curly brackets

Can luacode be used in the preamble of a standalone-type document?

Criticizing long fiction. How is it different from short?

What is Crew Dragon approaching in this picture?

4 Spheres all touching each other??

'A' vs 'an' in newspaper article

Predict mars robot position

Where is this triangular-shaped space station from?

How do you enable SQL Server 2019's result set caching?

How do I add a variable to this curl command?

Wanted: 5.25 floppy to usb adapter

Finding the number of integers that are a square and a cube at the same time

Is my plan for fixing my water heater leak bad?

How large should photos on my blog be?

Why do neural networks need so many training examples to perform?

How to satisfy a player character's curiosity about another player character?



Proof by Induction - New to proofs


Dominoes and induction, or how does induction work?Proving that $frac{phi^{400}+1}{phi^{200}}$ is an integer.Generating induction proofs from graphs/integralsMathematical induction proof; $g_k=3g_{k-1} - 2g_{k-2}$induction proof for kleene starTips on constructing a proof by induction.Induction proofs for subsets of integersproof using the mathematical inductionProof by Induction involving divisibilityInduction Proof:Inequality proof by induction(?)Proof by induction?













2












$begingroup$


totally new to proofs and found this challenge problem and struggling a bit. Any help would be appreciated!



There are some real numbers $x$ such that $x+frac{1}{x}$ is an
integer. For example, $2+sqrt{3}+frac{1}{2+sqrt{3}}=4$,
$1+frac{1}{1}=2$, and $2sqrt{6}-5+frac{1}{2sqrt{6}-5}=-10$.



Prove for all $xinmathbb{R}$ that if $x+frac{1}{x}$ is an integer,
then $x^n +frac{1}{x^n}$ also is an integer for all $ninmathbb{N}$.










share|cite|improve this question







New contributor




Robin is a new contributor to this site. Take care in asking for clarification, commenting, and answering.
Check out our Code of Conduct.







$endgroup$












  • $begingroup$
    I recommend this answer as a good start
    $endgroup$
    – Ross Millikan
    22 mins ago










  • $begingroup$
    math.stackexchange.com/questions/936479/…
    $endgroup$
    – lab bhattacharjee
    22 mins ago
















2












$begingroup$


totally new to proofs and found this challenge problem and struggling a bit. Any help would be appreciated!



There are some real numbers $x$ such that $x+frac{1}{x}$ is an
integer. For example, $2+sqrt{3}+frac{1}{2+sqrt{3}}=4$,
$1+frac{1}{1}=2$, and $2sqrt{6}-5+frac{1}{2sqrt{6}-5}=-10$.



Prove for all $xinmathbb{R}$ that if $x+frac{1}{x}$ is an integer,
then $x^n +frac{1}{x^n}$ also is an integer for all $ninmathbb{N}$.










share|cite|improve this question







New contributor




Robin is a new contributor to this site. Take care in asking for clarification, commenting, and answering.
Check out our Code of Conduct.







$endgroup$












  • $begingroup$
    I recommend this answer as a good start
    $endgroup$
    – Ross Millikan
    22 mins ago










  • $begingroup$
    math.stackexchange.com/questions/936479/…
    $endgroup$
    – lab bhattacharjee
    22 mins ago














2












2








2





$begingroup$


totally new to proofs and found this challenge problem and struggling a bit. Any help would be appreciated!



There are some real numbers $x$ such that $x+frac{1}{x}$ is an
integer. For example, $2+sqrt{3}+frac{1}{2+sqrt{3}}=4$,
$1+frac{1}{1}=2$, and $2sqrt{6}-5+frac{1}{2sqrt{6}-5}=-10$.



Prove for all $xinmathbb{R}$ that if $x+frac{1}{x}$ is an integer,
then $x^n +frac{1}{x^n}$ also is an integer for all $ninmathbb{N}$.










share|cite|improve this question







New contributor




Robin is a new contributor to this site. Take care in asking for clarification, commenting, and answering.
Check out our Code of Conduct.







$endgroup$




totally new to proofs and found this challenge problem and struggling a bit. Any help would be appreciated!



There are some real numbers $x$ such that $x+frac{1}{x}$ is an
integer. For example, $2+sqrt{3}+frac{1}{2+sqrt{3}}=4$,
$1+frac{1}{1}=2$, and $2sqrt{6}-5+frac{1}{2sqrt{6}-5}=-10$.



Prove for all $xinmathbb{R}$ that if $x+frac{1}{x}$ is an integer,
then $x^n +frac{1}{x^n}$ also is an integer for all $ninmathbb{N}$.







induction






share|cite|improve this question







New contributor




Robin is a new contributor to this site. Take care in asking for clarification, commenting, and answering.
Check out our Code of Conduct.











share|cite|improve this question







New contributor




Robin is a new contributor to this site. Take care in asking for clarification, commenting, and answering.
Check out our Code of Conduct.









share|cite|improve this question




share|cite|improve this question






New contributor




Robin is a new contributor to this site. Take care in asking for clarification, commenting, and answering.
Check out our Code of Conduct.









asked 31 mins ago









RobinRobin

261




261




New contributor




Robin is a new contributor to this site. Take care in asking for clarification, commenting, and answering.
Check out our Code of Conduct.





New contributor





Robin is a new contributor to this site. Take care in asking for clarification, commenting, and answering.
Check out our Code of Conduct.






Robin is a new contributor to this site. Take care in asking for clarification, commenting, and answering.
Check out our Code of Conduct.












  • $begingroup$
    I recommend this answer as a good start
    $endgroup$
    – Ross Millikan
    22 mins ago










  • $begingroup$
    math.stackexchange.com/questions/936479/…
    $endgroup$
    – lab bhattacharjee
    22 mins ago


















  • $begingroup$
    I recommend this answer as a good start
    $endgroup$
    – Ross Millikan
    22 mins ago










  • $begingroup$
    math.stackexchange.com/questions/936479/…
    $endgroup$
    – lab bhattacharjee
    22 mins ago
















$begingroup$
I recommend this answer as a good start
$endgroup$
– Ross Millikan
22 mins ago




$begingroup$
I recommend this answer as a good start
$endgroup$
– Ross Millikan
22 mins ago












$begingroup$
math.stackexchange.com/questions/936479/…
$endgroup$
– lab bhattacharjee
22 mins ago




$begingroup$
math.stackexchange.com/questions/936479/…
$endgroup$
– lab bhattacharjee
22 mins ago










1 Answer
1






active

oldest

votes


















6












$begingroup$

HINT: Note that for $ngeq1$ you have
$$left(x^n+frac{1}{x^n}right)left(x+frac{1}{x}right)=left(x^{n+1}+frac{1}{x^{n+1}}right)+left(x^{n-1}+frac{1}{x^{n-1}}right).$$



For more details, hover over the the block below:




The equation above can be rewritten to get
$$x^{n+1}+frac{1}{x^{n+1}}=left(x^n+frac{1}{x^n}right)left(x+frac{1}{x}right)-left(x^{n-1}+frac{1}{x^{n-1}}right).$$
If the three terms in parentheses on the right hand side are integers, then so is the left hand side. Now to use induction, all you need is that $x^n+frac{1}{x^n}$ is an integer for $n=0$ and $n=1$.







share|cite|improve this answer









$endgroup$













    Your Answer





    StackExchange.ifUsing("editor", function () {
    return StackExchange.using("mathjaxEditing", function () {
    StackExchange.MarkdownEditor.creationCallbacks.add(function (editor, postfix) {
    StackExchange.mathjaxEditing.prepareWmdForMathJax(editor, postfix, [["$", "$"], ["\\(","\\)"]]);
    });
    });
    }, "mathjax-editing");

    StackExchange.ready(function() {
    var channelOptions = {
    tags: "".split(" "),
    id: "69"
    };
    initTagRenderer("".split(" "), "".split(" "), channelOptions);

    StackExchange.using("externalEditor", function() {
    // Have to fire editor after snippets, if snippets enabled
    if (StackExchange.settings.snippets.snippetsEnabled) {
    StackExchange.using("snippets", function() {
    createEditor();
    });
    }
    else {
    createEditor();
    }
    });

    function createEditor() {
    StackExchange.prepareEditor({
    heartbeatType: 'answer',
    autoActivateHeartbeat: false,
    convertImagesToLinks: true,
    noModals: true,
    showLowRepImageUploadWarning: true,
    reputationToPostImages: 10,
    bindNavPrevention: true,
    postfix: "",
    imageUploader: {
    brandingHtml: "Powered by u003ca class="icon-imgur-white" href="https://imgur.com/"u003eu003c/au003e",
    contentPolicyHtml: "User contributions licensed under u003ca href="https://creativecommons.org/licenses/by-sa/3.0/"u003ecc by-sa 3.0 with attribution requiredu003c/au003e u003ca href="https://stackoverflow.com/legal/content-policy"u003e(content policy)u003c/au003e",
    allowUrls: true
    },
    noCode: true, onDemand: true,
    discardSelector: ".discard-answer"
    ,immediatelyShowMarkdownHelp:true
    });


    }
    });






    Robin is a new contributor. Be nice, and check out our Code of Conduct.










    draft saved

    draft discarded


















    StackExchange.ready(
    function () {
    StackExchange.openid.initPostLogin('.new-post-login', 'https%3a%2f%2fmath.stackexchange.com%2fquestions%2f3134247%2fproof-by-induction-new-to-proofs%23new-answer', 'question_page');
    }
    );

    Post as a guest















    Required, but never shown

























    1 Answer
    1






    active

    oldest

    votes








    1 Answer
    1






    active

    oldest

    votes









    active

    oldest

    votes






    active

    oldest

    votes









    6












    $begingroup$

    HINT: Note that for $ngeq1$ you have
    $$left(x^n+frac{1}{x^n}right)left(x+frac{1}{x}right)=left(x^{n+1}+frac{1}{x^{n+1}}right)+left(x^{n-1}+frac{1}{x^{n-1}}right).$$



    For more details, hover over the the block below:




    The equation above can be rewritten to get
    $$x^{n+1}+frac{1}{x^{n+1}}=left(x^n+frac{1}{x^n}right)left(x+frac{1}{x}right)-left(x^{n-1}+frac{1}{x^{n-1}}right).$$
    If the three terms in parentheses on the right hand side are integers, then so is the left hand side. Now to use induction, all you need is that $x^n+frac{1}{x^n}$ is an integer for $n=0$ and $n=1$.







    share|cite|improve this answer









    $endgroup$


















      6












      $begingroup$

      HINT: Note that for $ngeq1$ you have
      $$left(x^n+frac{1}{x^n}right)left(x+frac{1}{x}right)=left(x^{n+1}+frac{1}{x^{n+1}}right)+left(x^{n-1}+frac{1}{x^{n-1}}right).$$



      For more details, hover over the the block below:




      The equation above can be rewritten to get
      $$x^{n+1}+frac{1}{x^{n+1}}=left(x^n+frac{1}{x^n}right)left(x+frac{1}{x}right)-left(x^{n-1}+frac{1}{x^{n-1}}right).$$
      If the three terms in parentheses on the right hand side are integers, then so is the left hand side. Now to use induction, all you need is that $x^n+frac{1}{x^n}$ is an integer for $n=0$ and $n=1$.







      share|cite|improve this answer









      $endgroup$
















        6












        6








        6





        $begingroup$

        HINT: Note that for $ngeq1$ you have
        $$left(x^n+frac{1}{x^n}right)left(x+frac{1}{x}right)=left(x^{n+1}+frac{1}{x^{n+1}}right)+left(x^{n-1}+frac{1}{x^{n-1}}right).$$



        For more details, hover over the the block below:




        The equation above can be rewritten to get
        $$x^{n+1}+frac{1}{x^{n+1}}=left(x^n+frac{1}{x^n}right)left(x+frac{1}{x}right)-left(x^{n-1}+frac{1}{x^{n-1}}right).$$
        If the three terms in parentheses on the right hand side are integers, then so is the left hand side. Now to use induction, all you need is that $x^n+frac{1}{x^n}$ is an integer for $n=0$ and $n=1$.







        share|cite|improve this answer









        $endgroup$



        HINT: Note that for $ngeq1$ you have
        $$left(x^n+frac{1}{x^n}right)left(x+frac{1}{x}right)=left(x^{n+1}+frac{1}{x^{n+1}}right)+left(x^{n-1}+frac{1}{x^{n-1}}right).$$



        For more details, hover over the the block below:




        The equation above can be rewritten to get
        $$x^{n+1}+frac{1}{x^{n+1}}=left(x^n+frac{1}{x^n}right)left(x+frac{1}{x}right)-left(x^{n-1}+frac{1}{x^{n-1}}right).$$
        If the three terms in parentheses on the right hand side are integers, then so is the left hand side. Now to use induction, all you need is that $x^n+frac{1}{x^n}$ is an integer for $n=0$ and $n=1$.








        share|cite|improve this answer












        share|cite|improve this answer



        share|cite|improve this answer










        answered 28 mins ago









        ServaesServaes

        26.5k33997




        26.5k33997






















            Robin is a new contributor. Be nice, and check out our Code of Conduct.










            draft saved

            draft discarded


















            Robin is a new contributor. Be nice, and check out our Code of Conduct.













            Robin is a new contributor. Be nice, and check out our Code of Conduct.












            Robin is a new contributor. Be nice, and check out our Code of Conduct.
















            Thanks for contributing an answer to Mathematics Stack Exchange!


            • Please be sure to answer the question. Provide details and share your research!

            But avoid



            • Asking for help, clarification, or responding to other answers.

            • Making statements based on opinion; back them up with references or personal experience.


            Use MathJax to format equations. MathJax reference.


            To learn more, see our tips on writing great answers.




            draft saved


            draft discarded














            StackExchange.ready(
            function () {
            StackExchange.openid.initPostLogin('.new-post-login', 'https%3a%2f%2fmath.stackexchange.com%2fquestions%2f3134247%2fproof-by-induction-new-to-proofs%23new-answer', 'question_page');
            }
            );

            Post as a guest















            Required, but never shown





















































            Required, but never shown














            Required, but never shown












            Required, but never shown







            Required, but never shown

































            Required, but never shown














            Required, but never shown












            Required, but never shown







            Required, but never shown







            Popular posts from this blog

            Paper upload error, “Upload failed: The top margin is 0.715 in on page 3, which is below the required...

            Emraan Hashmi Filmografia | Linki zewnętrzne | Menu nawigacyjneGulshan GroverGulshan...

            How can I write this formula?newline and italics added with leqWhy does widehat behave differently if I...